Vous êtes sur la page 1sur 7

PHY481 - Outline of solutions to Homework 8

9.12. The free current is ~jf = j0 i for a z a. The magnetic susceptibility is m = 0


inside the slab and m finite outside the slab. Since there is a free current, the first step is
to find the magnetic intensity using H
H
~ d~l = if . Since the current is uniform, we expect

the field to be uniform. Using the right hand rule, the magnetic intensity is found to be
directed along the negative y-axis. The Amperian loop to consider is a rectangle with long
side l and short size 2z with normal along the x-axis, long sides parallel to the y-axis and
centered on the x-axis. We have to consider two cases z < a, z > a. For z < a the enclosed
free current is if = 2zlj0 , while for z > a the enclosed free current is if = 2alj0 . In both
cases, the contour integral gives 2Hl, so we find that

~ = zj0 j,
H a < z < a; ~ = aj0 j,
H ~ = aj0 j,
z>a H z < a (1)

~ = H
The magnetic field is then found from B ~ = 0 (1 + m )H,
~ so that,

~ = 0 zj0 j,
B a < z < a; ~ = 0 (1 + m )aj0 j,
H ~ = 0 aj0 j,
z>a H z < a (2)

~ = m H,
The magnetization is given by, M ~ so that,

~ = 0,
M a < z < a; ~ = m aj0 j,
M ~ = m aj0 j,
z>a M z < a (3)

~ b = M
The surface bound currents are then found from K ~ k at z = a, and K
~b = M~ k at

z = a. There are no bulk bound currents as ~ M ~ = 0 in all regimes where M~ is unifom.

The surface bound currents are then,

~ b = m aj0 i,
K z = a; ~ b = m aj0 i,
K z = a (4)

When the susceptibility is positive, the surface bound currents, Kb , are in the same
direction as the free current leading to an enhancement of the field in the material. When
the susceptibility is negative the surface bound currents are opposite the free current and
reduce the field inside the material.

9.15. Two long thin concentric conducting cylinders of radii a and b are separated by a
region of susceptibility m . The inner cylinder carries current I k, while the outer cylinder
carries current I k. Using H
H
~ d~l = if = 2rH for a circular contour, yields,

~ = 0,
H r < a; ~ = I a < r < b;
H ~ = 0,
H r>b (5)
2r

1
~ = m H,
The magnetization is found from M ~ so that,

~ = 0,
M r < a; ~ = m I a < r < b;
M ~ = 0,
M r>b (6)
2r
The bound currents are finite only in the regime a r b. There are surface bound
currents at r = a and at r = b, and we have to check for a bulk bound current in the regime
a < r < b as the magnetization is not constant. The surface bound currents are found from

K ~ r = m I k,
~b = M at r = b; and K ~ r = m I (k) at r = a
~b = M (7)
2b 2a
The bound currents are thus in opposite directions at a and b. However, when m > 0 both
of these bound currents produce a magnetic field inside the material that is larger than that
in the absence of the material. Using the expression for the curl in cylindrical co-ordinates
(see Table 2.3), the bulk bound currents are given by,
I
~b =
K ~ = 1 (r 2r ) = 0
~ M (8)
r r
Thus, even though the magnetization is not uniform the bulk bound current is zero.

9.17. a) Consider an infinite copper cylinder, with susceptibility m = 9.6 106 , and
~ = B0~k, with B0 = 1T at the surface of the cylinder. Define H0 = B0 /0 . There
with field B
are no free currents, so Htext = H0 = Htint . We then have,

~ int = 0 (1 + m )H
B ~ 0; ~ = m H0 k;
M ~b = M
K ~ r = m H0 sin (9)

b) Consider the cylinder with its axis along the z-direction and the field applied along the x-
axis. Since there are no free currents, we may take H ~ = ~ m . Gausss law for magnetism
~ B
~ = 0, along with the relation for a linear isotropic material B~ = H ~ = 0 (1 + m )H,
~
~ (H)
implies that ~ = 0. In regions of space or in parts of a material where is a constant,
~ H
we have ~ = 0. Using H
~ =
~ m in this equation leads to 2 m = 0. We may

thus solve the problem in a manner similar to that used for a dielectric sphere in a uniform
electric field, though the boundary conditions are different. The applied field is B0 , which
corresponds to H0 = B0 /0 . Assume that the solutions to Laplaces equations correspond to
a uniform field in the interior of the cylinder and a uniform field and a dipole in the exterior.
Note that this is a dipole in cylinderical co-ordinates (line dipole), so it has a different form
than that for point dipoles, we have (the radius of the cylinder is a),
a2
int
m = Ax = Arcos, r < a; ext
m = B0 rcos + C cos r > a (10)
r

2
Now we need two boundary conditions in order to solve for A and C. The first boundary
H
condition comes from the condition H~ d~l = if = 0, which implies that H ext H int = 0,
t t

as there are no free currents. The second condition comes from Gausss law for magnetism,
H
~ dA
B ~ = 0, so that Bnext Bnint = 0. These conditions imply that,

1 ext
m 1 int
m ext
m int
|a = |a ; and 0 |a = m |a ; (11)
r r r r
which lead to,
2H0 m H0
H0 = A + C; 0 (H0 + C) = A; solving gives A= ; C= (12)
m + 2 m + 2
Sometimes this is written in terms of m = /0 = m + 1. The magnetic intensity inside
and outside is then,
2
~ int = 2H0 ~ m H0 a cos)
~ ext = H0 i (
H i; H (13)
m + 2 m + 2 r
Using the expression for gradient in cylindrical co-ordinates gives,
2 2 2
~ m H0 a cos) = [ ( m H0 a cos)]r + [ 1 ( m H0 a cos)]
(
m + 2 r r m + 2 r r m + 2 r
m H0 a2
= (cosr + sinr) (14)
m + 2 r2
The dipole term in the external potential, ext
m is

m 0 H0 a2 ~ 0 r
0 m
cos = (15)
m + 2 r 2 r
~ 0 , is
so the line dipole moment (dipole moment per unit length), m
2m H0 a2
~0=
m (16)
m + 2
Alternatively we can use
2
~0=M
m ~ int a2 = 2m H0 a
~ a2 = m H (17)
m + 2
9.21. The magnetic moment of a current loop is m = iA. For a Bohr atom, mvr = h =
mr2 . The current is
q e eh
i= = = (18)
T 2 2mr2
The magnetic moment of the Bohr atom is then,
eh
m = r2 i = (19)
2m

3
The magnetization is the magnetic moment per unit volume (assuming they are all aligned),
so that
eh
M= (20)
2m(3 1010 )3
10.1 a) The motional emf is  = Blv = iR, so the current is Blv/R
b) The force is dF~ = id~l B,
~ which in this case reduces to a force of magnitude F = B 2 l2 v/R

in the direction opposite to the velocity.


c) Using Newtons equation, we have,

dv B 2 l2 v
= = R (21)
dt mR

where I defined = B 2 l2 /Rm. The solution to this equation, with initial velocity v0 is,
v(t) = v0 et . The distance travelled before stopping is
Z
v0 Rmv0
x= v0 et dt = = 22 (22)
0 B l

d) The power dissipated is i2 R, so the total energy dissipated is


Z
B 2 l2 v 2 Z 2 2
B l 2 2t
U= ( ) Rdt = v e dt (23)
0 R 0 R 0

Doing the integral yields U = mv02 /2, which is the amount of energy at the start.

10.2 The motional emf is Blv = 480kV

10.8 The square loop is at distance a from a long straight wire carrying current I0 cos(t).
The flux through the loop is given by,
Z
~ ~ 0 i(b a) Z b dr 0 i(b a)ln(b/a)
= B dA = = (24)
2 a r 2

The current induced in the loop is

dB 0 I0 sin(t)(b a)ln(b/a)
vsquare = = (25)
dt 2

and finally the current is vsquare /R.

10.20 The mutual inductance M in this problem is define through vloop = M diwire /dt or
N loop = M iwire , with the number of turns in the loop N = 1. The equalateral triangle has

4
a flat edge parallel to a wire and distance b from the wire. The apex of the triangle is at
distanc a + b from the wire. The flux in the loop is then given by,
Z l Z y
0 idy 0
loop = 2 dx (26)
0 0 2(b + y 0 )

where l = a/31/2 , which is one half the length of a side and y = xtan60 = 31/2 x. Evaluating
give, loop = M i, with
0 b a a a
M= 1/2
[(1 + )ln(1 + ) ] (27)
3 b b b
10.27 Energy = volume B 2 /(20 ).
10.28 The magnetic field of a dipole is,
~ r)r m
~ = 0 3(m ~
B (28)
4 r3
and the field energy density is given by, u = B 2 /20 , so the total magnetic energy is given
by,
Z Z
Z
3 1 0 2
U= ud r = 2 r2 sin ( ~ r)r m)
) (3(m ~ 3(m
~ r)r m
~ (29)
0 RE 0 4r3
which gives,
Z Z
Z
3 1 0 2 2
U= ud r = 2 r2 sin ( 3
) m (3cos2 + 1) (30)
0 RE 0 4r
or
Z
3 0 m2 Z Z 1 2 0 m2
U= ud r = sin(3cos + 1) = 3
(31)
8 2 0 RE r4 12RE
11.2 a) Plates radius a, separation d with potential difference V (t). Using a circular
contour and
~ d~l = 0 i + 0 0 dE
Z
B (32)
dt
The LHS gives 2B(r). The current enclosed is zero. For r < a, the electric flux enclosed
in the loop is E = V r2 /d, while for r > a, the enclosed electric flux is V a2 /d. We then
find that,
0 0 r dV 0 0 a2 dV
B(r) = ; r < a, B(r) = ; r>a (33)
2d dt 2rd dt
b) A wire carrying i has field 0 i/(2r). The current is dQ/dt which is also CdV /dt,
where C = 0 A/d = 0 a2 /d. Combining these equations, gives the result quoted.

11.13 Maxwells equations in differential form, in vaccum, are given by,

~ E
~ = 0, (34)

5
~ B
~ =0 (35)
~
~ E
~ = B (36)
t
~
~ B
~ = 0 0 E (37)
t
To check that the solutions that are given satisfy these four equations, use Cartesian co-
ordinates. The electric field is given by

Ex = 0, Ey = 0, Ez = E0 cos(x/L)cos(y/L)sin(t) (38)

~ is given by,
and the divergence of E

Ex Ey Ez
+ + =0 (39)
x y z

The magnetic field is

Bx = B0 cos(x/L)sin(y/L)cos(t), By = B0 sin(x/L)cos(y/L)cos(t) Bz = 0
(40)
~ is given by,
so the divergence of B

Bx By B0 B0
+ +0= sin(x/L)sin(y/L)cos(t) sin(x/L)sin(y/L)cos(t) = 0
x y L L
(41)
The other parts are similar plug and chug
11.21 a) Intensity is the average energy per unit area per unit time. The energy per
unit time per unit area or energy flux density is related to the electric and magnetic fields
through the Poynting vector
~= 1E
S ~ B
~ (42)
0
The electric and magnetic fields are perpendicular to each other and in phase so the cross
product gives
1
P eak energy f lux density = Epeak Bpeak (43)
mu0
The peak energy flux density is twice the intensity, and the peak magnetic and electric fields
are related by, Epeak = cBpeak , so we have,

1
1300W/m2 = cB02 so that B0 = 2.3 106 T ; E0 = 700V /m (44)
20

6
Similar calculations for b) and c)

d) n(t)h = Intensity where n(t) is the number of photons arriving per unit area per
unit time.

Vous aimerez peut-être aussi